Storing Cookies (See : http://ec.europa.eu/ipg/basics/legal/cookies/index_en.htm ) help us to bring you our services at overunity.com . If you use this website and our services you declare yourself okay with using cookies .More Infos here:
https://overunity.com/5553/privacy-policy/
If you do not agree with storing cookies, please LEAVE this website now. From the 25th of May 2018, every existing user has to accept the GDPR agreement at first login. If a user is unwilling to accept the GDPR, he should email us and request to erase his account. Many thanks for your understanding

User Menu

Custom Search

Author Topic: A possible violation of the Law of Conservation of Energy  (Read 49846 times)

Zetetic

  • Newbie
  • *
  • Posts: 33
A possible violation of the Law of Conservation of Energy
« on: April 14, 2015, 10:59:57 PM »
Hello.

I thought you guys might be interested in this.

It seems to me that you guys have two hurdles to overcome; one theoretical and the other practical.  To successfully build a free energy device you must, one, find an exception to the Law of Conservation of Energy and, two, then get a device mechanically to work with this exception.

I believe I have found a flaw in the Law of Conservation of Energy.  Unfortunately, I don’t think this particular flaw I found can lead to a mechanical free energy device.  But, having said that, if I’m right and I found an exception to this law, and thus cleared the theoretical hurdle, I thought you guys still might be interested in this.

I posted my argument at Science Forums.  The original post of my argument was really long and I don’t think anyone really read it fully.  The comments that followed were all tangential and didn’t address the issue I raised.

I kept thinking about the question and was able to get it down to two pages.  I posted this new formulation of the question in the same thread on April 10th.  There were no responses.  But I know they were reading it based on the “view counter” increasing.  I tried bating them into commenting (e.g. “Silence, generally, means consent”).  And finally today, April 14th, I got one response.  But it was meaningless.  The comment was not that there was a logical or factual flaw in my question but rather that had a “poor use of fonts.”  And this, to me, seems like another concession that they can’t find an actual flaw in my argument.

So, anyway, I thought you guys might be interested in this.  Here is the link to post #21 where I propose the shorted and sharper argument:

http://www.scienceforums.net/topic/87489-a-magnet-is-demagnetized/page-2

If any of you have ever posted anything on Science Forums you know that they love to make snarky comments putting people like you and me in our place when we question the “known Laws of Physics.”  Their silence is, I believe, very telling.  It makes me think that I may really be onto something.  I may be wrong (I am all the time), but right now I think I really may have found something.

Again, I don’t think it leads the mechanics needed to make an actual free energy device.  But, maybe it does and one of you can find it.  I don’t know.

If you do go and read this discussion at Science Forums and if you then choose to comment, please be careful.  If I’m right and I’ve found a violation of one of their most fundamental beliefs, and that’s why they can’t respond, I’m sure they would love to lock up the thread and end my blasphemy.  I’ve been very careful to try to formulate everything in the form of a question and to try to avoid anything that would violate their rules.  All they need is for me or someone else to make a declarative statement like “physics is flawed” or “this proves the Law of Conservation of Energy is wrong” to lock the thread up and move it off to the dustbin of their “Speculations” folder.

So, it’s not a free energy device, but I thought it might be related enough to the stuff you guys are into that you might be interested.

And that’s it.  (Again, I may be wrong but right now I think I really may have found something.)

Take care.



ayeaye

  • Hero Member
  • *****
  • Posts: 866
Re: A possible violation of the Law of Conservation of Energy
« Reply #1 on: April 16, 2015, 05:36:19 PM »
I'm likely not into all the scientific subtleties of this, and what laws of physics it may or may not violate. Going into equations would take aeons there i guess, so i prefer simple principles. Thus sorry if my question is somewhat naive. So what are you going to do, achieve a continuous movement by demagnetizing a permanent magnet using heat? This may well be possible, but then does not heating the magnet take energy?

There may be overunity in permanent magnets, but by my experiments i found that this is not enough for continuous rotation. Overunity is not the same as permanent motion. But can you say where the energy on the image below comes from? I don't say that the law of conservation of energy is violated, i just don't know where the energy comes from. And for making  free energy devices, i also don't see that it is important to answer to that question. I did the experiment of that, see this video https://archive.org/details/Flcm3 . This experiment was done just so that one can repeat it, and see oneself whether the results are the same. Just make sure to use magnets of the same size and strength, and a disc with a good ball bearings.

The result was that when starting to move from almost the same point, the disc moved some two times more by the chain of magnets, than in the opposite direction. This should not be so when all the energy came from moving the disc to the starting position, and all the movement was only caused by repulsion of the first magnet. This does not provide a continuous rotation with permanent magnets or electromagnets, so for that it is useless. Which does not mean that such effect may not cause overunity when charged particles move in a solid state device.

Thank you for your attention.

Zetetic

  • Newbie
  • *
  • Posts: 33
Re: A possible violation of the Law of Conservation of Energy
« Reply #2 on: April 16, 2015, 09:40:30 PM »

hey ayeaye ...

Thanks for the video link.  Cool design!

I watched the video you linked to (Field lines chain motor 3/4), as well as the links to the previous video (Field lines chain motor 1/4) and the next video (Field lines chain motor 4/4).

(Is there also a “Field lines chain motor 2/4” and I just missed it?)

The “Field lines chain motor 1/4” worked as I would have guess it would.  Whether you have one magnet that is capable of moving or several aligned magnets capable of moving, I would expect them to move so that the pole of the moveable magnet (or magnets) moves away from where it is repulsed by the external and fixed magnet and to where it is attracted to the external and fixed magnet (the one in your hand).

I was surprised by the results of “Field lines chain motor 3/4.”  I did not expect the movable magnets to spin past, and to spin so far past, the end of the chain where the pole of the moveable magnets is attracted to the external and fixed in place magnet.  That was interesting.

What my guess is that you have found a magnetic arrangement, in “Field lines chain motor 3/4”, where the repulsed starting point is a greater force than the attractive point at the end of the row (curved row) of magnets.  And so, even though the row of magnets are attracted to the external fixed magnet (as was seen in “Field lines chain motor 1/4”) in this arrangement the force and the resultant velocity from the repulsed starting point is enough to break free and move beyond the greatest point of attraction between the moveable magnets and the external fixed magnet in “Field lines chain motor 3/4”.  So, I was surprised by the results, but it makes sense that such a configuration as you found could be made.

“Field lines chain motor 4/4” did work as I would have expected.  By going from a row of magnets with a south end and a north end (“Field lines chain motor 3/4”) to a full circle (“Field lines chain motor 4/4”) means going from a system where there is a point of greater repulsion and a point of greater attraction to a system where there is no point of greater repulsion and no point of greater attraction (assuming a perfect physical system with no imperfections).  And what I would expect in this case is, after the moveable magnets have been set in motion (by your hand), for them to not continue in motion by a force from the arrangement (since there is no greater point of repulsion for them to get away from and no greater point of attraction for them to move towards), but rather move for a while due to having been set in motion by your hand and then eventually come to a stop due to friction (your device, like all real world devices, is not frictionless and friction brings motion to a stop (while generating thermal energy).)  And the fact that your device in “Field lines chain motor 4/4” tended to come to a stop at the same point in the circle of magnets also is what I would have expected.  In a hypothetical perfect circle of magnets I would expect it to come to a stop at different random points.  However, in a real world device (where there will always been some slight differences, imperfections, in the manufactured magnets and some slight differences (even if only very very tiny) in the size and shape of the gaps between the magnets) it is expected that there would be some point around the circle of magnets where the attraction of the circle of magnets is greater (even if only slightly greater) with the external fixed magnet.

I know the results of “Field lines chain motor 3/4,” where the curved row of magnets moved well beyond the external fixed magnet, might seem like this would lead to continuous motion forever when then moving from this configuration to a complete circle of magnets, but I think that by going from a row to a complete circle you are eliminating the dynamic (a point of greater repulsion and a point of greater attraction) that caused the movement in “Field lines chain motor 3/4.”

I think that if you were able to physically build a truly perfect “Field lines chain motor 4/4” (with no slight imperfections or variations) that it would come to a stop, due to friction, at random points, while if you were to build such a design (with real world materials and real world placements that will always be something less, even if only slightly less, that perfect) then you’d see it come to a stop in one or two (or so) places more often than not.

But ... this is just my opinion.

---

If you’re interested in the Law of Conservation of Energy and how it relates to free energy devices, I can explain it to you in a couple of very short and common sense language paragraphs (no Physics jargon needed, no math needed, and I promise it will only be a small fraction of the length of my review of you design).

So, if you’re up for that, brief, discussion/explanation ... let me know.

---

Again, cool design!  Thank you for showing it to me.  I hope my analysis of it is of some use.

Take care!

- Zet 



(PS:  While I believe I understood what you were going for in the videos, I’m sorry, but I don’t think I quite understood what you were trying to explain with your drawing.)




ayeaye

  • Hero Member
  • *****
  • Posts: 866
Re: A possible violation of the Law of Conservation of Energy
« Reply #3 on: April 16, 2015, 11:06:54 PM »
What i was trying to explain with my drawing is this. A north pole of another magnet is moving through the field of the magnet, which is as it is drawn there. And it moves in the direction shown by the arrow. Now when this pole of the magnet is where the field lines are horizontal, then it moves by the field lines, this is what field lines mean. And when moving by the field lines, it gets a propulsion, to the direction where these field lines go.

The only thing which may work against it, is a repulsion, that is going through the field lines when their direction is opposite to the movement. But as can be seen on the drawing, this never happens, because in the worst case the field lines are perpendicular to the movement, and never opposite to the movement. Which means that the moving magnet gets extra speed when moving through the magnetic field of the other magnet, and thus an additional energy. This should not happen when there is no overunity, the pole of the magnet should go through the field of another magnet without getting any additional energy. Thus the additional energy should come from somewhere, the question is from where.

The reason why this happens is that magnetic field is not symmetric. It were symmetric if it were just radial lines coming from the magnet, like a field around a sphere. But it is not like that, and thus magnetic field is not symmetric. And every asymmetric field can in theory do work.

Zetetic

  • Newbie
  • *
  • Posts: 33
Re: A possible violation of the Law of Conservation of Energy
« Reply #4 on: April 17, 2015, 01:28:40 AM »



ayeaye,


I guess I’m not sure how you have the two magnets oriented towards one another in the drawing.

In your videos I thought your magnets (the set moving magnets (row and full circle) and the fixed magnet) were orientated relative one another as they are in A in the drawing below.  (But, of course, in your videos it’s the other magnet or magnets that is in motion and the other magnet that is fixed.)

And so, this is how I thought they were also oriented in your drawing.

But, now, based on your last response, I think maybe in your drawing you have the orientation different, as shown in B in the drawing below.

I hope to understand what you’re getting at, but we need to clear up my understanding of your point.

A?
B?
Or, something else?


- Zet




ayeaye

  • Hero Member
  • *****
  • Posts: 866
Re: A possible violation of the Law of Conservation of Energy
« Reply #5 on: April 17, 2015, 02:18:49 AM »
It is always A. In my experiment, the upper magnet in A is the stator magnet, and the lower magnet in A is one of the rotor magnets. The stator magnet is a kind of long, so only one pole, the north pole, really matters, the effect of the other pole is very small and can be disregarded. On my drawing, the upper magnet in A is shown as moving, and the lower magnet is shown as standing still.

I used a long cylindrical neodymium magnet as the stator magnet. It is important in that experiment that the stator magnet is strong, and the rotor magnets are small. Mainly because the rotor is lighter then.

So the upper magnet is really the stator magnet, but it is a kind of opposite on the drawing, i showed it so because i thought that it's better to explain it that way. As it doesn't really matter which one of the magnets stands still and which one moves, because it only matters that the magnets move relative to each other.

Zetetic

  • Newbie
  • *
  • Posts: 33
Re: A possible violation of the Law of Conservation of Energy
« Reply #6 on: April 17, 2015, 03:41:49 AM »


ayeaye,



Okay.  Your magnets are always orientated like in drawing A in reply #4.  That makes sense.  Thank you for clearing that up for me.

I now have a second basic question about your drawing.  Hopefully we can also clear that up for me and then I can finally get to understanding and then addressing the point you are making with your drawing.

In your drawing you show the moving magnet before reaching the fixed magnet and then you show the moving magnet after passing the fixed magnet.  And, when the magnets are in those positons, you indicate that there is “no repulsion.”

I think I get your point, but I think the logic needs to be fleshed out a little bit more.

When the magnets are in position X (in the drawing below) there is no repulsion (so I agree with your drawing about that), but there is attraction (which is not indicated in your drawing).  And this attraction inhibits the leftward movement of the moving magnet.

And when the magnets are in position Y (in the drawing below) there is no “repulsion” in the sense of pushing the magnet rightward.  However, when the magnets are in position Y there is both attraction pulling the magnet leftward and (I believe) repulsion pushing the magnet leftward.

This is counterintuitive.  It seems as if when the moving magnet is to the right of the fixed magnet that the moving magnet should be repulsed to the right.  However, the magnetic lines of flux, obviously, extend out beyond the physical location of actual magnet itself.  And so, when the moving magnet is just slightly to the right of the fixed magnet there are repulsive magnetic lines of flux to the right of the moving magnet and this will push the moving magnet leftward.  It counterintuitive, but if you think about it it makes sense, and it’s one of those weird aspects of magnetic interaction that I find fascinating.

But if the magnets are in position Z (in the drawing below), if the moving magnet is just slightly more to the right of the fixed magnet, then there is repulsion which will push the moving magnet rightward (and this is not represented in your drawing).

This can be seen in your video “Field lines chain motor 1/4” at about 1 minute and 21 seconds.  You place the fixed magnet at the start slightly more to the right than in the other iterations and the moving magnets (in this case it is the other magnets that are the moveable) are repulsed away.

So, again, I’m trying to understand the basics of your drawing before trying to understand the point you are making with it.

There is no repulsion at point X (as you indicate in your drawing), but there is also attraction (which is not included in your drawing).

There is no “repulsion” at point Y (as you indicate in your drawing) in the sense of pushing the moving magnet rightwards (but there is a leftward repulsion).

And there is repulsion when the moving magnet is to the right of the fixed magnet, point Z, but this occurs when the moving magnet is slightly more to the right of the fixed magnet than point Y (and this is not indicated in your drawing).

No?

Did I get something wrong?

Let me know!

Again, it’s a cool design.  Thanks for sharing it!


- Zet

(PS:  I agree with you that it doesn’t matter which is the fixed magnet and which is the moving magnet (a difference between your videos and your drawing) but only relative the orientation between the two matters.)



ayeaye

  • Hero Member
  • *****
  • Posts: 866
Re: A possible violation of the Law of Conservation of Energy
« Reply #7 on: April 17, 2015, 05:23:25 AM »
The pole of the magnet moves by field lines. What concerns the repulsion in the beginning and the attraction in the end, then this happens only when the field lines go from a pole of the standing (lower) magnet to the pole of the moving magnet. This is insignificant when the lower magnet is a part of a chain of magnets. Then the field lines from its south pole go to the north pole of the previous magnet, and the field lines from its north pole go to the south pole of the next magnet.

This drawing was mostly drawn with the field lines chain design in mind. With a stand alone magnet some field lines certainly go to the pole of the moving (upper) magnet. Then it may just depend on how long are the field lines. Whether there is any overunity in that case, i don't know, this drawing was to explain the results of the field lines chain experiment.
« Last Edit: April 17, 2015, 08:45:01 AM by ayeaye »

ayeaye

  • Hero Member
  • *****
  • Posts: 866
Re: A possible violation of the Law of Conservation of Energy
« Reply #8 on: April 17, 2015, 06:03:47 PM »
So the concept basically is, that the magnetic field is an asymmetric field, so in some circumstances it can do work. And my experiments seem to confirm this. Yet it cannot provide permanent motion at least in a mechanical way.

OK, tell about your thing, what is the reason why it can be overunity?

Zetetic

  • Newbie
  • *
  • Posts: 33
Re: A possible violation of the Law of Conservation of Energy
« Reply #9 on: April 17, 2015, 07:56:25 PM »


I thought I was getting a better handle on your proposal, but now I’m not so sure.  I feel like maybe I’ve taken one step forward and two steps back (in trying to understand it).

I’m trying!

Based on what you wrote in Reply #7 I have yet another basic question.

There is a concept in the physics of magnetics called the “unmagnetized” state.  “Unmagnetized” is different from “demagnetized.”  Here is a link to a description of the former (if you’re interested):

http://www.irm.umn.edu/hg2m/hg2m_d/hg2m_d.html

It’s somewhat technical, but I think it may be relevant to what you are working on.

When a magnet is “demagnetized” the poles of the particles within it become randomized and so there is no longer an external magnetic field and thus the effects of the field are gone.  And when a magnet becomes “unmagnetized” (as a ferromagnet has the natural tendency to do) the magnet moves to a state with more and more multiple poles within the single body, and this reigns in the external magnetic field and so here also the effects of the external field are also (essentially) gone.

I’ve never heard of someone making a ring of magnets in order to reign in the external magnetic field.  But in your Reply #7 it seems like you might be suggesting that that is what you are doing.  And, again while I’ve never heard of someone doing this, it make sense that your ring of magnets would essentially lead to a state of “unmagnetization” (in the sense of the word used in the link above).

But, if this is what you are suggesting, then I’m confused.  This would then mean the “unmagnetized” ring would not significantly interact with the external fixed in place magnet.  And isn’t the interaction between the ring of magnets and the external fixed magnet the whole point of your design?

Again, I’m sorry.  I’m trying!



---



Okay, back to the idea in my original post.  Thanks for asking!

The link in the original post does not lead to an argument for an Overunity Device but rather to an argument for why the Law of Conservation of Energy is false.  And the two things, while different, are related.  And I can explain this in very simple terms.

The Law of Conservation of Energy says that while energy can change forms the total amount of energy remains the same.

There are different kinds of energy.  Motion is a form of energy (“kinetic energy”).  Heat is a form of energy (“thermal energy”).  A ball some distance above the surface of the Earth is a form of energy (“potential energy”).  And so on.

This “Law” says that energy can change from one form to another (e.g. motion can become heat and heat can become motion) but the amount of energy remains the same.

So, for example, say I roll a ball across the floor.  When the ball leaves my hand it is in motion, and this motion is a certain amount of energy, and let’s say its 10 units of kinetic energy.  As the ball rolls across the floor it eventually comes to a stop.  The 10 units of kinetic energy is now gone.  What brings the ball to a stop is the friction between the ball and the floor.  And friction generates heat.  And so, when the ball comes to a stop, there is a loss of 10 units of kinetic energy but there is also an equal increase of 10 units of thermal energy (heat).

And so, in the ball rolling across the floor example, energy has changed forms (motion becomes heat) but the total amount of energy remains the same (10 units of kinetic energy becomes 10 units of thermal energy).



Okay, why is this relevant to OU?

Say someone was successful at making a working device that once set in motion continued moving forever (such as if you were able to get “Field lines chain motor” to spin forever).  There is a certain amount of energy put into the system at the start.  It takes energy for your hand to originally set it into motion (there is a decrease in biological energy and an equal increase in kinetic energy in the form of the spinning disk).  Now, say, it keeps moving.  This would mean there is no decrease in kinetic energy.  But, as it moves there is friction (between the spinning disk and the bearings and the axel and between the spinning disk and the surrounding air and so on).  And friction generates heat (thermal energy).

So, if your design (or anyone else’s in this forum) was to keep moving after being set into motion, despite encountering friction, there would be no decrease in kinetic energy while all the time it is moving more and more thermal energy would be generated.  Energy would be created!  The total amount of energy within the Universe would increase!

And this would be a violation of the Law of Conservation of Energy.  The total amount of energy would not remain the same.  There would be no decrease in kinetic energy while there is an increase thermal energy.

(There is also the quest for a perpetual motion machine of the second kind which does not violate the Law of Conservation of Energy but rather violates the Second Law of Thermodynamics, but that’s a different topic.)



So, back to my original post (and the argument it links to).

I’ve spent years playing around with trying to build a perpetual motion machine (free energy device, overunity device).  But I’ve also spent years just playing around with the logic of the Law of Conservation of Energy itself.

And I believe I’ve been able to craft an argument that shows a violation of the Law of Conservation of Energy, which means, if I’m right, shows the Law of Conservation of Energy is false.

Unfortunately, the specific violation (and thus disproof) of this “Law” that I found does not also (I don’t think) lead to the mechanics needed to design and build an OU device.

But, if I’m right, and I have shown this “Law” to be false, this would validate the efforts of you and me and everyone else in this forum.  If you were to show your “Field lines chain motor” to an actual physicist, he or she wouldn’t waste any time looking at the mechanics of it to see if you’ve got something, they would “know” that there must be a flaw in there somewhere because the Law of Conservation of Energy tells them that such a device as yours could never work and so they’d just dismiss it out of hand without any examination.

If I’m right and I’ve found a flaw in (and thus disproved) the Law of Conservation of Energy in my argument (linked to in the OP), and if this could become established and well known, then efforts like yours in your design would be given credibility in the “established” scientific community.

And that’s it.

That’s the point of my post here.

If you read my argument in the link in the original post, let me know what you think!

(However, if you want to talk more about the mechanics of your design I’m happy to do so ... I would enjoy doing so!)



- Zet


(PS:  Oops.  I promised in my Reply #2 to explain the Law of Conservation of Energy and its relation to OU in a couple of short paragraphs.  Oops.  I guess I went a little bit overboard here.  Sorry.)



ayeaye

  • Hero Member
  • *****
  • Posts: 866
Re: A possible violation of the Law of Conservation of Energy
« Reply #10 on: April 17, 2015, 10:34:01 PM »
I just did experiments, to see what happens. Yes i also tried to make a full circle of magnets but this, maybe indeed became what you say "unmagnetized". That is, the poles of some magnets shifted, for some unknown reason. And this is why the continuous rotation was not possible.

The experiments showed overunity only with shorter chains of magnets, something changed whenever it was made to full circle.

My hand, all you have to do is to move the disc near the top of the peak near the first magnet. You feel it, and you have to move it as close to that peak as possible. Then it moves in different directions depending on to what side of the peak you moved it. Your hand of course provides energy, but if you move it near the peak every time, the energy you provided should be the same both when the disc moves in one direction, or in the direction opposite to it. Now what shows overunity is that it moves some two times more in one direction, than in the opposite direction, in spite that the initial energy you provided was the same.

I did it with my hand, you can try it with your hand, and if the magnets and the disc are well repeated, you should get the same results. I know that its better to use mechanical things, and not hand, as hand is not very reliable, and the video cannot show all the slight movements of the hand, which could affect the result. But the experiment is simpler and thus much more easily repeatable that way. Because what i also know is that no experiment can be provable, experiments can only be repeatable. Thus easy repeatability is better than more provability. So i made the experiment as simple and as easily repeatable as possible.

I used a computer's processor fan for bearings, but i oiled it well. And i used only mounting tape for fixing things. Tesa, scotch, 3m, whatever you get works well. But magnets are a bigger problem, i don't know where to get so small ceramic magnets which i used in the video 3/4. I got them from some small decorations which they put on the refrigerator door, but not all of them have so small magnets. One may try very small neodymium cube magnets, which can be bought, but i'm not sure whether the result will be the same.

What concerns the law of conservation of energy, then i think it is in the very essence just an expression of the balance in the universe. So even if you can show that it is not valid in one process, it is valid more generally. But no matter what the theories, conservation of energy or the energy came from some unknown source, i don't want to speculate where the energy came from. Because if i did, and if it happens that my guess was wrong, then for that same reason it would be considered that all that i said was wrong.

Zetetic

  • Newbie
  • *
  • Posts: 33
Re: A possible violation of the Law of Conservation of Energy
« Reply #11 on: April 18, 2015, 04:36:36 AM »

ayeaye,



Thank you for continuing the conversation on Conservation of Energy.

“So even if you can show that it is not valid in one process, it is valid more generally.”    -ayeaye

This is the crux of the issue.

The Law of Conservation of Energy is a “Law” and not a “Principle.”

Mainstream Physics does not claim that energy is “generally” conserved.  It claims that energy is always conserved.  They claim that no one can ever “show that it is not valid in one process.”

If they are right, then all the attempts at creating an Overunity Device as seen in this forum are nothing more than a waste of time.  A successful Overunity Device would be a mechanical device that violates the Law of Conservation of Energy, and if they are right, then no Overunity Device attempt will ever succeed.

This is the issue.  And this is the point of me posting a link to my disproof of this “Law” in this forum.

And so it’s my hope and you, or anyone else in this forum who disagrees with mainstream Physics and believes an Overunity Device is possible (and therefore necessarily believes the “Law of Conservation of Energy” is not a “Law” but a “Principle”), might then discuss with me whether or not my argument (linked to in the OP) does actually disprove this “Law.”

That’s it.  That’s my purpose in posting here.


---


I think I’ve figured out why you get the results you get in “Field lines chain motor 3/4.”


Think of a hypothetical frictionless pendulum (see Q in the drawing below).

The pendulum starts out at rest and at a certain height (Q.1).  It then swings down and increases in velocity until it reaches its lowest point (Q.2).  And then it swings back up and decelerates until it finally comes to a stop and at the same height from which it started (Q.3).

The velocity reached at its lowest point is the exact amount of speed needed for it to then reach the same height on the other side (again, in a hypothetical frictionless world).

The same thing can be done with a magnet on a rotating disk and an external fixed magnet (see R in the drawing below).  Again, this is in a hypothetically frictionless world.  The two magnets (the moveable one and the fixed one) are orientated so that their dissimilar poles, which are attracted to one another, will pass by one another facing one another.

The moveable magnet starts at a rest at a certain distance from the fixed magnet (R.1).  It then rotates towards the fixed magnet and increases in velocity until it reaches the fixed magnet (R.2).  And then it continues to rotate beyond the fixed magnet and decelerates until it finally comes to a stop at the same distance from the fixed magnet from which it started (R.3).

The velocity reached when it is at the point of the fixed magnet is the exact amount of speed needed for it to then reach the same distance from the fixed magnet on the other side (in a hypothetical frictionless world).

Now the same thing is done again, but with the magnets orientated relative to one another as they are in “Field lines chain motor 3/4.”

The moveable magnet starts out at rest just to the left of the fixed magnet where it will be repulsed to the right and not to the left (S.1).  But at this point, and this is the key point, the moveable magnet is both repulsed and attracted to the fixed magnet.  In this position, the right end North Pole of the moveable magnet is repulsed by the North Pole of the fixed magnet and the left end South Pole of the moveable magnet is attracted to the North Pole of the fixed magnet.  The moveable magnet then rotates to the left and increases in velocity until it reaches the point of greatest attraction between the moveable and fixed magnets (S.2).  It increases in velocity due to both repulsion and attraction.  It then continues to rotate beyond the point of greatest attraction and decelerates due to the attraction between the moveable magnet and the fixed magnet.  It decelerates, and again this is part of the key point, due now only to attraction (S.3).

The velocity it reaches when at the point of greatest attraction is from repulsion and attraction and so will be greater than the velocity needed to escape and move beyond the attraction (and only the attraction) while it is decelerating on the other side.

This is why, I think, you get the results you do in 3/4.  (And, even though you’ve taken steps to reduce the friction as much as possible, in your real world model (as in all real world models) there is friction and this is my guess why it then finally does come to a stop after escaping the attraction with the fixed magnet.)

So, you and I may disagree about the reason why for the results you get.  I hope I haven’t offended you with an alternative theory.  Maybe I’m wrong.  But this is what makes sense to me right now as I look at it.  I’m happy to continue to discuss it with you and even to try to find flaws in my analysis!  (I’m interested in truth in the end and I’m not interested in “winning” arguments in the passing.)

Again, I hope you don’t mind me offering an alternative theory.

Take care!

- Zet




(PS:  I’ve never built your exact model in “3/4” but I’ve built similar kinds of models.  I, too, have “felt” the “peak” of the magnetic repulsion.  I’ve always found this a fun and fascinating experience!)





ayeaye

  • Hero Member
  • *****
  • Posts: 866
Re: A possible violation of the Law of Conservation of Energy
« Reply #12 on: April 18, 2015, 06:25:18 AM »
My drawing was about one magnet in a chain of magnets, and i have only seen overunity with a chain of magnets. Ultimately the friction stops the movement of course. But overunity is because the repulsion of the first magnet of the chain should be the greatest propulsion there, if the other magnets in the chain do not add additional energy. But because the disc moves some two times more by the chain, they evidently do, the way the drawing shows.

The energy necessary to move the disc to the peak of the magnetic force at the first magnet in the chain, is also equal to the energy released by the repulsion of that magnet. When the disc moves some two times more than that by the chain of magnets, the energy released is more than the input energy. The known input energy that is, because the only known input energy is the energy necessary for moving the disc to the peak of the magnetic force.

allcanadian

  • Hero Member
  • *****
  • Posts: 1317
Re: A possible violation of the Law of Conservation of Energy
« Reply #13 on: April 18, 2015, 08:33:37 AM »
@Zetetic
Quote
If any of you have ever posted anything on Science Forums you know that they love to make snarky comments putting people like you and me in our place
when we question the “known Laws of Physics.” Their silence is, I believe, very telling.  It makes me think that I may really be onto something.  I may be wrong (I am all the time), but right now I think I really may have found something.
Again, I don’t think it leads the mechanics needed to make an actual free energy device.  But, maybe it does and one of you can find it.  I don’t know.
You make a very good case and I had to think on it for a while. I believe the conservation of energy relates to the known laws of physics however one can never know all of the near infinite number of ways the law(s) may be applied.
You know within a few minutes of reading your post I may have found a way to prove the paradox. Now imagine we had a very strong permanent magnet insulated from a hot metal plate in a box by a near perfect insulator such as aerogel. If we had metal balls inside the box then they would be attracted upward to the magnet however they would first impact the metal plate generating heat and this heat would partially demagnetize them causing them to fall to the bottom and repeat the cycle. Obviously the metal plate must get hotter from these impacts regardless of whether it was preheated to start the process or not. As well the impact in itself is also know to disturb the magnetic domains only increasing the effect. We could also pull a vacuum in the box to reduce the heat loss from top to bottom. Now if the bottom of the box dissipated part of the heat generated by the impacts cooling the balls just below the Curie temperature then we would theoretically have Maxwells demon.
 
The metal balls would in effect be the Demon generating heat through the transformation of kinetic energy to thermal energy on impact which in effect maintains their motion. However we should note the transformation is simply the macroscopic kinetic energy reduced to kinetic energy acting on a smaller scale we call heat. This also relates to a statment by Faraday-- "It does not matter how the change occurs only that it does". At which point we might ask would this be overunity? Well no because the universal radiation field induced the charge on the fundamental particles in matter which caused them to move which created a displacement field we call magnetism which was then disturbed by the thermal energy created on impact. The supposed extra energy would still be real energy transferred from the external field to make up the difference it's just that we never thought of that. Overunity is simply a convienient term to describe the fact we don't understand what the hell is going on, lol.
Would this ill concieved conservation breaking demon machine work in reality?, most likely not however is does raise some interesting questions.
AC
 
 

Zetetic

  • Newbie
  • *
  • Posts: 33
Re: A possible violation of the Law of Conservation of Energy
« Reply #14 on: April 18, 2015, 09:43:17 PM »


allcanadian,



Thank you for reading my argument against (hopefully my “disproof of”) the Law of Conservation of Energy.

-

“I believe the conservation of energy relates to the known laws of physics however one can never know all of the near infinite number of ways the law(s) may be applied.” – allcanadian

Are you saying that the Law of Conservation of Energy is generally true but not absolutely and always true (as ayeaye seemed to suggest above)?  If so, I agree with both of you guys.  But this means we are all in disagreement with mainstream physics.  And that’s the point of my argument in Science Forums.

My argument (linked to in the OP) is a very simple piece of logic that (hopefully) shows that there is a flaw in the logic of the Law of Conservation of Energy.  And this means we can know that this “Law” is false.  And we can know that it is false to a logical certainty!  (No experiments needed.)

And this, I would think, would be something of interest to people who are trying to build mechanical devices where more energy is outputted than is imputed (which, if successful, would be a violation of the Law of Conservation of Energy, and where, alternatively, if this “Law” is in fact true, then these devices could never really work).

That’s it.

(If anyone does read my argument linked to in the OP, please feel free to just read the question and ignore or skim over the endnotes.  The question is very straight forward and simple, but the endnotes, at times, are a bit tedious.  The endnotes are there simply in anticipation of the likely issues to be raised in response to my argument.  They are superfluous.)

-

Before I comment on your two balls in a box example, I want to make sure I understand the set up.  In drawing M below I have visually described what I think you are saying.  There’s a good chance I’m wrong, so please correct me if so.  That’s the point of me drawing it out: to make sure I understand your set up.  And then, once we are on the same page, I’d love to comment on it and discuss it with you.

However, please note (as I’m sure you know) that Maxwell’s Demon is a Second Law of Thermodynamics issue and not a Law of Conservation of Energy issue.  Both “Laws,” and the hopeful disproof of both “Laws,” are relevant to people like us who want to see successful perpetual motion machines.  But they are two different and distinct theories, and the mechanical devices that purport to violate them are two different and distinct kinds of mechanical devices.

If my argument linked to in the OP tends to inspire a Second Law of Thermodynamics analysis then, my bad, it should not have.  There is a “heat pack” in that argument.  But it is only there for the purpose of demagnetizing (raising above the Curie temperature) the magnets, and therefore ending the mutual attraction between the two magnets.  The thermal energy generated does nothing else in that thought experiment.  It does not create more or less (macro) kinetic energy.  My argument in the link in the OP is a disproof (hopefully) of the Law of Conservation of Energy and not an attempted disproof of the Second Law of Thermodynamics (although I suspect the latter must also have exceptions, just not in the argument I made, too).

I hope that’s clear.  If not, please let me know.

So, while I suspect your two balls in a box example is beyond the scope of what I originally posted in this thread, I’m interested.  I’d like to understand it and discuss it.  So, please, review my drawings below and see if I’ve understood or not your proposal.  And, then, we can talk about the specifics of it.

Take care!

---


ayeaye,


I understand that in your “3/4” you have a row (a cured row) of multiple magnets and in my drawing S in Reply #11 I show only one magnet.  But I think the dynamics would be just the same.

Consider, for the ease of analysis, a straight magnet and a straight row of magnets with no gaps and a straight row of magnets with gaps (drawing N below).

In all three cases, the greatest point is repulsion is on the right end and the greatest point of attraction is on the left end.  In the case of the single bar magnet there are no other points of repulsion or attraction in between.  In the case of row of magnets with no gaps, theoretically, there are no points of repulsion or attraction in between (if all the magnets are perfectly aligned, but in the real world there will always be some physical imperfections and so there will be some (very minor) repulsions and attractions in between).  And in the case of the row of magnets with gaps there will definitely be points of repulsion and attraction in between, but these will be minor relative to the repulsion at the right end of the row and the attraction at the left end of the row.

(I make the claims that I do in the above paragraph based on years and years of playing around with magnetic configurations like this in an attempt to find a dynamic that would then lead to a successful perpetual motion machine.  I never did.)

If I understand you correctly, it is your proposal that with the gaps in in “3/4” each individual magnet in the row gives a separate push on the disk as they pass by the external fixed magnet.  It is my contention that the gaps, while they do have some impact on the system, do not push the disk further along.  It is my contention that whether the gaps are present or whether you did the same thing with one magnets or with a row of magnets with no gaps, you would get the same results as you do with the gaped row (curved gaped row) of magnets in “3/4.”  It is my contention that if you have one magnet the same (or similar) length as the row of gapped magnets that you’d get the same (or similar) distance clearance in the end in both cases (and that it is not the individual separated magnets that each give the moving body a separate and additional push, but rather the repulsion and attraction at the right and left ends of both systems).

Again, I hope you don’t mind me offering an alternative theory for your results in “3/4.”  We are on, and beyond, the borderlines of accepted science and so we should expect (just as there is within the borderlines of accepted science) lots of disagreements!  I love your design.  I never tried building your exact model.  But I did work with similar ideas.  (And perhaps you can find the exception, if there is one, in this area that I was never able to find!)

Take care.

---


- Zet